Correct Way of Reaching Inductive Goal












0












$begingroup$


To provide full context of the practice question I'm attempting, it is as follows:




For every integer n such that n ≥ 8, there exist nonnegative integers an and bn such that 3an + 5bn = n.



Write a proof of the claim using the strong form of mathematical induction with the integer 10 as breakpoint — such that that n = 8, n = 9 and n = 10 would all be considered in the basis.




This is my attempt at the proof, but I can't seem to get past applying the I.H. in order to reach my goal in the inductive step.




Basis

If n = 8, then 3a8 + 5b8 = 8

To satisfy the above, set a8 = 1 and b8 = 1, therefore 3(1) + 5(1) = 3 + 5 = 8



If n = 9, then 3a9 + 5b9 = 9

Set a9 = 3 and b9 = 0, therefore 3(3) + 5(0) = 9 + 0 = 9



If n = 10, then 3a10 + 5b10 = 10



Set a10 = 0 and b10 = 2, therefore 3(0) + 5(2) = 0 + 10 = 10



Inductive Hypothesis

Let k be an integer such that k ≥ 10. It is necessary and sufficient to use the following:




  • Inductive Hypothesis: 3an + 5bn = n for every integer n such that 8 ≤ n ≤ k.


Inductive Claim
3ak+1 + 5bk+1 = k+1



Since for every integer n, 8 ≤ n ≤ k, there are ak-2 and bk-2, then it follows that:



3ak-2 + 5bk-2 = k - 2 (Starting Point)



..



..



..



3ak+1 + 5bk+1 = k+1 (Goal)




Now I know where to begin and the goal I need to reach for my inductive step but I am unsure as to how to reach my goal.



Any guidance would be much appreciated!










share|cite|improve this question











$endgroup$












  • $begingroup$
    Am I missing something? Is it not sufficient to set $a_n = a_{n-3}+1$ and $b_n = b_{n-3}$?
    $endgroup$
    – Brian Tung
    Jan 15 at 18:21






  • 2




    $begingroup$
    You can have a look at For all non-negative integers $n$ such that $n ≥ 8$, there exist non-negative integers $a_n$ and $b_n$ such that $3a_n+5b_n = n$. Found using Approach0. More tips on searching: How to search on this site?
    $endgroup$
    – Martin Sleziak
    Jan 15 at 18:23










  • $begingroup$
    Simply 3 + 5 = 8, 3×3 = 9, 5x2 = 10 is sufficient to establish the base step. Toss in a 5×0 or a 3×0 if you'd like.
    $endgroup$
    – William Elliot
    Jan 15 at 21:33
















0












$begingroup$


To provide full context of the practice question I'm attempting, it is as follows:




For every integer n such that n ≥ 8, there exist nonnegative integers an and bn such that 3an + 5bn = n.



Write a proof of the claim using the strong form of mathematical induction with the integer 10 as breakpoint — such that that n = 8, n = 9 and n = 10 would all be considered in the basis.




This is my attempt at the proof, but I can't seem to get past applying the I.H. in order to reach my goal in the inductive step.




Basis

If n = 8, then 3a8 + 5b8 = 8

To satisfy the above, set a8 = 1 and b8 = 1, therefore 3(1) + 5(1) = 3 + 5 = 8



If n = 9, then 3a9 + 5b9 = 9

Set a9 = 3 and b9 = 0, therefore 3(3) + 5(0) = 9 + 0 = 9



If n = 10, then 3a10 + 5b10 = 10



Set a10 = 0 and b10 = 2, therefore 3(0) + 5(2) = 0 + 10 = 10



Inductive Hypothesis

Let k be an integer such that k ≥ 10. It is necessary and sufficient to use the following:




  • Inductive Hypothesis: 3an + 5bn = n for every integer n such that 8 ≤ n ≤ k.


Inductive Claim
3ak+1 + 5bk+1 = k+1



Since for every integer n, 8 ≤ n ≤ k, there are ak-2 and bk-2, then it follows that:



3ak-2 + 5bk-2 = k - 2 (Starting Point)



..



..



..



3ak+1 + 5bk+1 = k+1 (Goal)




Now I know where to begin and the goal I need to reach for my inductive step but I am unsure as to how to reach my goal.



Any guidance would be much appreciated!










share|cite|improve this question











$endgroup$












  • $begingroup$
    Am I missing something? Is it not sufficient to set $a_n = a_{n-3}+1$ and $b_n = b_{n-3}$?
    $endgroup$
    – Brian Tung
    Jan 15 at 18:21






  • 2




    $begingroup$
    You can have a look at For all non-negative integers $n$ such that $n ≥ 8$, there exist non-negative integers $a_n$ and $b_n$ such that $3a_n+5b_n = n$. Found using Approach0. More tips on searching: How to search on this site?
    $endgroup$
    – Martin Sleziak
    Jan 15 at 18:23










  • $begingroup$
    Simply 3 + 5 = 8, 3×3 = 9, 5x2 = 10 is sufficient to establish the base step. Toss in a 5×0 or a 3×0 if you'd like.
    $endgroup$
    – William Elliot
    Jan 15 at 21:33














0












0








0





$begingroup$


To provide full context of the practice question I'm attempting, it is as follows:




For every integer n such that n ≥ 8, there exist nonnegative integers an and bn such that 3an + 5bn = n.



Write a proof of the claim using the strong form of mathematical induction with the integer 10 as breakpoint — such that that n = 8, n = 9 and n = 10 would all be considered in the basis.




This is my attempt at the proof, but I can't seem to get past applying the I.H. in order to reach my goal in the inductive step.




Basis

If n = 8, then 3a8 + 5b8 = 8

To satisfy the above, set a8 = 1 and b8 = 1, therefore 3(1) + 5(1) = 3 + 5 = 8



If n = 9, then 3a9 + 5b9 = 9

Set a9 = 3 and b9 = 0, therefore 3(3) + 5(0) = 9 + 0 = 9



If n = 10, then 3a10 + 5b10 = 10



Set a10 = 0 and b10 = 2, therefore 3(0) + 5(2) = 0 + 10 = 10



Inductive Hypothesis

Let k be an integer such that k ≥ 10. It is necessary and sufficient to use the following:




  • Inductive Hypothesis: 3an + 5bn = n for every integer n such that 8 ≤ n ≤ k.


Inductive Claim
3ak+1 + 5bk+1 = k+1



Since for every integer n, 8 ≤ n ≤ k, there are ak-2 and bk-2, then it follows that:



3ak-2 + 5bk-2 = k - 2 (Starting Point)



..



..



..



3ak+1 + 5bk+1 = k+1 (Goal)




Now I know where to begin and the goal I need to reach for my inductive step but I am unsure as to how to reach my goal.



Any guidance would be much appreciated!










share|cite|improve this question











$endgroup$




To provide full context of the practice question I'm attempting, it is as follows:




For every integer n such that n ≥ 8, there exist nonnegative integers an and bn such that 3an + 5bn = n.



Write a proof of the claim using the strong form of mathematical induction with the integer 10 as breakpoint — such that that n = 8, n = 9 and n = 10 would all be considered in the basis.




This is my attempt at the proof, but I can't seem to get past applying the I.H. in order to reach my goal in the inductive step.




Basis

If n = 8, then 3a8 + 5b8 = 8

To satisfy the above, set a8 = 1 and b8 = 1, therefore 3(1) + 5(1) = 3 + 5 = 8



If n = 9, then 3a9 + 5b9 = 9

Set a9 = 3 and b9 = 0, therefore 3(3) + 5(0) = 9 + 0 = 9



If n = 10, then 3a10 + 5b10 = 10



Set a10 = 0 and b10 = 2, therefore 3(0) + 5(2) = 0 + 10 = 10



Inductive Hypothesis

Let k be an integer such that k ≥ 10. It is necessary and sufficient to use the following:




  • Inductive Hypothesis: 3an + 5bn = n for every integer n such that 8 ≤ n ≤ k.


Inductive Claim
3ak+1 + 5bk+1 = k+1



Since for every integer n, 8 ≤ n ≤ k, there are ak-2 and bk-2, then it follows that:



3ak-2 + 5bk-2 = k - 2 (Starting Point)



..



..



..



3ak+1 + 5bk+1 = k+1 (Goal)




Now I know where to begin and the goal I need to reach for my inductive step but I am unsure as to how to reach my goal.



Any guidance would be much appreciated!







discrete-mathematics proof-verification induction






share|cite|improve this question















share|cite|improve this question













share|cite|improve this question




share|cite|improve this question








edited Jan 15 at 18:18









Martin Sleziak

45.1k10123277




45.1k10123277










asked Jan 15 at 17:44









man2006man2006

175




175












  • $begingroup$
    Am I missing something? Is it not sufficient to set $a_n = a_{n-3}+1$ and $b_n = b_{n-3}$?
    $endgroup$
    – Brian Tung
    Jan 15 at 18:21






  • 2




    $begingroup$
    You can have a look at For all non-negative integers $n$ such that $n ≥ 8$, there exist non-negative integers $a_n$ and $b_n$ such that $3a_n+5b_n = n$. Found using Approach0. More tips on searching: How to search on this site?
    $endgroup$
    – Martin Sleziak
    Jan 15 at 18:23










  • $begingroup$
    Simply 3 + 5 = 8, 3×3 = 9, 5x2 = 10 is sufficient to establish the base step. Toss in a 5×0 or a 3×0 if you'd like.
    $endgroup$
    – William Elliot
    Jan 15 at 21:33


















  • $begingroup$
    Am I missing something? Is it not sufficient to set $a_n = a_{n-3}+1$ and $b_n = b_{n-3}$?
    $endgroup$
    – Brian Tung
    Jan 15 at 18:21






  • 2




    $begingroup$
    You can have a look at For all non-negative integers $n$ such that $n ≥ 8$, there exist non-negative integers $a_n$ and $b_n$ such that $3a_n+5b_n = n$. Found using Approach0. More tips on searching: How to search on this site?
    $endgroup$
    – Martin Sleziak
    Jan 15 at 18:23










  • $begingroup$
    Simply 3 + 5 = 8, 3×3 = 9, 5x2 = 10 is sufficient to establish the base step. Toss in a 5×0 or a 3×0 if you'd like.
    $endgroup$
    – William Elliot
    Jan 15 at 21:33
















$begingroup$
Am I missing something? Is it not sufficient to set $a_n = a_{n-3}+1$ and $b_n = b_{n-3}$?
$endgroup$
– Brian Tung
Jan 15 at 18:21




$begingroup$
Am I missing something? Is it not sufficient to set $a_n = a_{n-3}+1$ and $b_n = b_{n-3}$?
$endgroup$
– Brian Tung
Jan 15 at 18:21




2




2




$begingroup$
You can have a look at For all non-negative integers $n$ such that $n ≥ 8$, there exist non-negative integers $a_n$ and $b_n$ such that $3a_n+5b_n = n$. Found using Approach0. More tips on searching: How to search on this site?
$endgroup$
– Martin Sleziak
Jan 15 at 18:23




$begingroup$
You can have a look at For all non-negative integers $n$ such that $n ≥ 8$, there exist non-negative integers $a_n$ and $b_n$ such that $3a_n+5b_n = n$. Found using Approach0. More tips on searching: How to search on this site?
$endgroup$
– Martin Sleziak
Jan 15 at 18:23












$begingroup$
Simply 3 + 5 = 8, 3×3 = 9, 5x2 = 10 is sufficient to establish the base step. Toss in a 5×0 or a 3×0 if you'd like.
$endgroup$
– William Elliot
Jan 15 at 21:33




$begingroup$
Simply 3 + 5 = 8, 3×3 = 9, 5x2 = 10 is sufficient to establish the base step. Toss in a 5×0 or a 3×0 if you'd like.
$endgroup$
– William Elliot
Jan 15 at 21:33










1 Answer
1






active

oldest

votes


















0












$begingroup$

3 + 5 = 8, 3×3 = 9, 5x2 = 10



The next step is 11 = 8 + 3.

Since 8 has been answered, an answer for 11 is apparent.

Likewise for n > 10, by induction hypothesis,

there is a solution for n - 3, so a solution for n.



Exercise. Show every integer >= 8 is a multiple of 3

or 5 + a multiple of 3 or 10 + a multiple of 3

as an immediate result of the above proof.






share|cite|improve this answer









$endgroup$














    Your Answer








    StackExchange.ready(function() {
    var channelOptions = {
    tags: "".split(" "),
    id: "69"
    };
    initTagRenderer("".split(" "), "".split(" "), channelOptions);

    StackExchange.using("externalEditor", function() {
    // Have to fire editor after snippets, if snippets enabled
    if (StackExchange.settings.snippets.snippetsEnabled) {
    StackExchange.using("snippets", function() {
    createEditor();
    });
    }
    else {
    createEditor();
    }
    });

    function createEditor() {
    StackExchange.prepareEditor({
    heartbeatType: 'answer',
    autoActivateHeartbeat: false,
    convertImagesToLinks: true,
    noModals: true,
    showLowRepImageUploadWarning: true,
    reputationToPostImages: 10,
    bindNavPrevention: true,
    postfix: "",
    imageUploader: {
    brandingHtml: "Powered by u003ca class="icon-imgur-white" href="https://imgur.com/"u003eu003c/au003e",
    contentPolicyHtml: "User contributions licensed under u003ca href="https://creativecommons.org/licenses/by-sa/3.0/"u003ecc by-sa 3.0 with attribution requiredu003c/au003e u003ca href="https://stackoverflow.com/legal/content-policy"u003e(content policy)u003c/au003e",
    allowUrls: true
    },
    noCode: true, onDemand: true,
    discardSelector: ".discard-answer"
    ,immediatelyShowMarkdownHelp:true
    });


    }
    });














    draft saved

    draft discarded


















    StackExchange.ready(
    function () {
    StackExchange.openid.initPostLogin('.new-post-login', 'https%3a%2f%2fmath.stackexchange.com%2fquestions%2f3074701%2fcorrect-way-of-reaching-inductive-goal%23new-answer', 'question_page');
    }
    );

    Post as a guest















    Required, but never shown

























    1 Answer
    1






    active

    oldest

    votes








    1 Answer
    1






    active

    oldest

    votes









    active

    oldest

    votes






    active

    oldest

    votes









    0












    $begingroup$

    3 + 5 = 8, 3×3 = 9, 5x2 = 10



    The next step is 11 = 8 + 3.

    Since 8 has been answered, an answer for 11 is apparent.

    Likewise for n > 10, by induction hypothesis,

    there is a solution for n - 3, so a solution for n.



    Exercise. Show every integer >= 8 is a multiple of 3

    or 5 + a multiple of 3 or 10 + a multiple of 3

    as an immediate result of the above proof.






    share|cite|improve this answer









    $endgroup$


















      0












      $begingroup$

      3 + 5 = 8, 3×3 = 9, 5x2 = 10



      The next step is 11 = 8 + 3.

      Since 8 has been answered, an answer for 11 is apparent.

      Likewise for n > 10, by induction hypothesis,

      there is a solution for n - 3, so a solution for n.



      Exercise. Show every integer >= 8 is a multiple of 3

      or 5 + a multiple of 3 or 10 + a multiple of 3

      as an immediate result of the above proof.






      share|cite|improve this answer









      $endgroup$
















        0












        0








        0





        $begingroup$

        3 + 5 = 8, 3×3 = 9, 5x2 = 10



        The next step is 11 = 8 + 3.

        Since 8 has been answered, an answer for 11 is apparent.

        Likewise for n > 10, by induction hypothesis,

        there is a solution for n - 3, so a solution for n.



        Exercise. Show every integer >= 8 is a multiple of 3

        or 5 + a multiple of 3 or 10 + a multiple of 3

        as an immediate result of the above proof.






        share|cite|improve this answer









        $endgroup$



        3 + 5 = 8, 3×3 = 9, 5x2 = 10



        The next step is 11 = 8 + 3.

        Since 8 has been answered, an answer for 11 is apparent.

        Likewise for n > 10, by induction hypothesis,

        there is a solution for n - 3, so a solution for n.



        Exercise. Show every integer >= 8 is a multiple of 3

        or 5 + a multiple of 3 or 10 + a multiple of 3

        as an immediate result of the above proof.







        share|cite|improve this answer












        share|cite|improve this answer



        share|cite|improve this answer










        answered Jan 15 at 21:48









        William ElliotWilliam Elliot

        9,2662820




        9,2662820






























            draft saved

            draft discarded




















































            Thanks for contributing an answer to Mathematics Stack Exchange!


            • Please be sure to answer the question. Provide details and share your research!

            But avoid



            • Asking for help, clarification, or responding to other answers.

            • Making statements based on opinion; back them up with references or personal experience.


            Use MathJax to format equations. MathJax reference.


            To learn more, see our tips on writing great answers.




            draft saved


            draft discarded














            StackExchange.ready(
            function () {
            StackExchange.openid.initPostLogin('.new-post-login', 'https%3a%2f%2fmath.stackexchange.com%2fquestions%2f3074701%2fcorrect-way-of-reaching-inductive-goal%23new-answer', 'question_page');
            }
            );

            Post as a guest















            Required, but never shown





















































            Required, but never shown














            Required, but never shown












            Required, but never shown







            Required, but never shown

































            Required, but never shown














            Required, but never shown












            Required, but never shown







            Required, but never shown







            Popular posts from this blog

            Cabo Verde

            Gyllenstierna

            Karlovacs län